subject
Mathematics, 04.08.2019 07:30 chellybean

Using the data from table 8.3, what is the portfolio expected return and the portfolio beta if you invest 35 percent in x, 45 percent in y, and 20 percent in the riskminus−free asset? a. 12.5%, 1.975 b. 15.0%, 1.975 c. 15.0%, 0.975 d. 12.5%, 0.975

ansver
Answers: 1

Another question on Mathematics

question
Mathematics, 20.06.2019 18:04
For an experiment, a penny is tossed. at the same time, a spinner with 8 congruent sections numbered 1–8 is spun. how many possible outcomes are there for the experiment?
Answers: 1
question
Mathematics, 21.06.2019 13:30
12. decide whether each statement is always true, sometimes true, or never true. a. when corresponding points of an original figure and its reflection are connected, the resulting segments are all perpendicular to the line of reflection. b. when a point is reflected over a horizontal line, the y-coordinate of the point stays the same. answer both questions and explain
Answers: 2
question
Mathematics, 21.06.2019 15:30
Which is the completely factored from of xy^3-x^3y
Answers: 3
question
Mathematics, 21.06.2019 17:00
Simply 8x squared + 5(7x squared- x) - 2x =
Answers: 1
You know the right answer?
Using the data from table 8.3, what is the portfolio expected return and the portfolio beta if you i...
Questions
question
Mathematics, 24.05.2021 15:10
question
Social Studies, 24.05.2021 15:10
Questions on the website: 13722367